Factors in a different base $ 2b^2!+!9b!+!7,mid, 7b^2!+!9b!+!2$












6















Two numbers $297_B$ and $792_B$, belong to base $B$ number system. If the first number is a factor of the second number, then what is the value of $B$?




Solution:
enter image description here



But base cannot be negative. Could someone please explain where I am going wrong?










share|cite|improve this question





























    6















    Two numbers $297_B$ and $792_B$, belong to base $B$ number system. If the first number is a factor of the second number, then what is the value of $B$?




    Solution:
    enter image description here



    But base cannot be negative. Could someone please explain where I am going wrong?










    share|cite|improve this question



























      6












      6








      6


      2






      Two numbers $297_B$ and $792_B$, belong to base $B$ number system. If the first number is a factor of the second number, then what is the value of $B$?




      Solution:
      enter image description here



      But base cannot be negative. Could someone please explain where I am going wrong?










      share|cite|improve this question
















      Two numbers $297_B$ and $792_B$, belong to base $B$ number system. If the first number is a factor of the second number, then what is the value of $B$?




      Solution:
      enter image description here



      But base cannot be negative. Could someone please explain where I am going wrong?







      elementary-number-theory divisibility number-systems






      share|cite|improve this question















      share|cite|improve this question













      share|cite|improve this question




      share|cite|improve this question








      edited 9 hours ago









      greedoid

      37.9k114794




      37.9k114794










      asked 10 hours ago









      Aamir Khan

      455




      455






















          4 Answers
          4






          active

          oldest

          votes


















          10














          The long division is the source of the error; you can't have $7/2$ as the quotient. The quotient needs to be an integer, that's what "factor" means.



          If the quotient is $2$, then the base is $4$. This is found by solving $7B^2+9B+2=color{red}{ 2}(2B^2+9B+7)$, and discarding the negative root.



          If the quotient is $3$, then the base is $19$. This is found by solving $7B^2+9B+2=color{red}{ 3}(2B^2+9B+7)$, and discarding the negative root.



          No other quotients make any sense. However, if the base is $4$, then you don't get digits $7$ and $9$. Hence the answer must be $B=19$.






          share|cite|improve this answer





















          • Thank you so much. This was really very helpful. :)
            – Aamir Khan
            10 hours ago










          • My pleasure, glad to help.
            – vadim123
            10 hours ago










          • @AamirKhan Beware that the above is not a rigorous solution without proof that those are the only possible quotients. See my answer for one rigorous approach
            – Bill Dubuque
            10 hours ago








          • 1




            @BillDubuque, it's not difficult to make it rigorous, to prove that the quotient $q$ can't be bigger than $3$. $(2q-7)B^2+(9q-9)B+(7q-2)$ is strictly positive if $qge 4$ and $B>0$. Hence if $qge 4$, no positive $B$ solves the equation. Similarly, if $q=1$, then the only positive solution is $B=1$, which is not possible.
            – vadim123
            9 hours ago












          • @vadim You should add the details of a rigorous proof to the answer (I've lost count of the number of times "[this case] doesn't make sense" turned out to be incorrect), so we should not encourage students to handwave like that.
            – Bill Dubuque
            9 hours ago





















          6














          Going $1$ step more with Euclid's algorithm reveals a common factor $,b!+!1.,$ Cancelling it



          $$dfrac{7b^2!+!9b!+!2}{2b^2!+!9b!+!7} = color{#c00}{dfrac{7b!+!2}{2b!+!7}}inBbb Z , Rightarrow, 7-2 dfrac{color{#c00}{7b!+!2}}{ color{#c00}{2b!+!7}}, =, dfrac{45}{2b!+!7}inBbb Zqquad$$



          Therefore $,2b!+!7mid 45 $ so $,b> 9,$(= digit) $,Rightarrow,2b!+!7 = 45,$ $Rightarrow,b=19.$






          share|cite|improve this answer































            1














            Since $b+1>0$ and $$(b+1)(2b+7)mid (7b+2)(b+1)implies 2b+7mid 7b+2$$



            we have $$2b+7mid (7b+2)-3(2b+7) = b-19$$



            so if $b-19> 0$ we have $$2b+7mid b-19 implies 2b+7leq b-19 implies b+26leq 0$$



            which is not true. So $bleq 19$. By trial and error we see that $b=4$ and $b=19$ works.






            share|cite|improve this answer























            • This answer is of no use?
              – greedoid
              7 hours ago



















            0














            $$2B^2+9B+7mid 7B^2+9B+2$$



            Let's write $aB^2+bB + c$ as $[a,b,c]_B$ to emphasis that $a,b,c$ are digits base $B$.



            Then $[2,9,7]_B mid [7,9,2]_B-[2,9,7]_B$ and we are assuming that $2,9,7 < B$



            Writing this out "subtraction-style", we get



            $left.begin{array}{c}
            7 & 9 & 2 \
            2 & 9 & 7 \
            hline
            phantom{4}
            end{array}
            right.
            implies
            left.begin{array}{c}
            6 & (B+8) & (B+2) \
            2 & 9 & 7 \
            hline
            4 & (B-1) & (B-5)
            end{array}
            right.
            $



            So $[4,B-1,B-5]_B$ is a multiple of $[2,9,7]_B$.



            We must therefore have $[4,B-1,B-5]_B = 2[2,9,7]_B = [4,18,14]_B$ which implies $B-1=18$ and $B-5=14$. Hence $B=19$.






            share|cite|improve this answer























              Your Answer





              StackExchange.ifUsing("editor", function () {
              return StackExchange.using("mathjaxEditing", function () {
              StackExchange.MarkdownEditor.creationCallbacks.add(function (editor, postfix) {
              StackExchange.mathjaxEditing.prepareWmdForMathJax(editor, postfix, [["$", "$"], ["\\(","\\)"]]);
              });
              });
              }, "mathjax-editing");

              StackExchange.ready(function() {
              var channelOptions = {
              tags: "".split(" "),
              id: "69"
              };
              initTagRenderer("".split(" "), "".split(" "), channelOptions);

              StackExchange.using("externalEditor", function() {
              // Have to fire editor after snippets, if snippets enabled
              if (StackExchange.settings.snippets.snippetsEnabled) {
              StackExchange.using("snippets", function() {
              createEditor();
              });
              }
              else {
              createEditor();
              }
              });

              function createEditor() {
              StackExchange.prepareEditor({
              heartbeatType: 'answer',
              autoActivateHeartbeat: false,
              convertImagesToLinks: true,
              noModals: true,
              showLowRepImageUploadWarning: true,
              reputationToPostImages: 10,
              bindNavPrevention: true,
              postfix: "",
              imageUploader: {
              brandingHtml: "Powered by u003ca class="icon-imgur-white" href="https://imgur.com/"u003eu003c/au003e",
              contentPolicyHtml: "User contributions licensed under u003ca href="https://creativecommons.org/licenses/by-sa/3.0/"u003ecc by-sa 3.0 with attribution requiredu003c/au003e u003ca href="https://stackoverflow.com/legal/content-policy"u003e(content policy)u003c/au003e",
              allowUrls: true
              },
              noCode: true, onDemand: true,
              discardSelector: ".discard-answer"
              ,immediatelyShowMarkdownHelp:true
              });


              }
              });














              draft saved

              draft discarded


















              StackExchange.ready(
              function () {
              StackExchange.openid.initPostLogin('.new-post-login', 'https%3a%2f%2fmath.stackexchange.com%2fquestions%2f3058019%2ffactors-in-a-different-base-2b2-9b-7-mid-7b2-9b-2%23new-answer', 'question_page');
              }
              );

              Post as a guest















              Required, but never shown

























              4 Answers
              4






              active

              oldest

              votes








              4 Answers
              4






              active

              oldest

              votes









              active

              oldest

              votes






              active

              oldest

              votes









              10














              The long division is the source of the error; you can't have $7/2$ as the quotient. The quotient needs to be an integer, that's what "factor" means.



              If the quotient is $2$, then the base is $4$. This is found by solving $7B^2+9B+2=color{red}{ 2}(2B^2+9B+7)$, and discarding the negative root.



              If the quotient is $3$, then the base is $19$. This is found by solving $7B^2+9B+2=color{red}{ 3}(2B^2+9B+7)$, and discarding the negative root.



              No other quotients make any sense. However, if the base is $4$, then you don't get digits $7$ and $9$. Hence the answer must be $B=19$.






              share|cite|improve this answer





















              • Thank you so much. This was really very helpful. :)
                – Aamir Khan
                10 hours ago










              • My pleasure, glad to help.
                – vadim123
                10 hours ago










              • @AamirKhan Beware that the above is not a rigorous solution without proof that those are the only possible quotients. See my answer for one rigorous approach
                – Bill Dubuque
                10 hours ago








              • 1




                @BillDubuque, it's not difficult to make it rigorous, to prove that the quotient $q$ can't be bigger than $3$. $(2q-7)B^2+(9q-9)B+(7q-2)$ is strictly positive if $qge 4$ and $B>0$. Hence if $qge 4$, no positive $B$ solves the equation. Similarly, if $q=1$, then the only positive solution is $B=1$, which is not possible.
                – vadim123
                9 hours ago












              • @vadim You should add the details of a rigorous proof to the answer (I've lost count of the number of times "[this case] doesn't make sense" turned out to be incorrect), so we should not encourage students to handwave like that.
                – Bill Dubuque
                9 hours ago


















              10














              The long division is the source of the error; you can't have $7/2$ as the quotient. The quotient needs to be an integer, that's what "factor" means.



              If the quotient is $2$, then the base is $4$. This is found by solving $7B^2+9B+2=color{red}{ 2}(2B^2+9B+7)$, and discarding the negative root.



              If the quotient is $3$, then the base is $19$. This is found by solving $7B^2+9B+2=color{red}{ 3}(2B^2+9B+7)$, and discarding the negative root.



              No other quotients make any sense. However, if the base is $4$, then you don't get digits $7$ and $9$. Hence the answer must be $B=19$.






              share|cite|improve this answer





















              • Thank you so much. This was really very helpful. :)
                – Aamir Khan
                10 hours ago










              • My pleasure, glad to help.
                – vadim123
                10 hours ago










              • @AamirKhan Beware that the above is not a rigorous solution without proof that those are the only possible quotients. See my answer for one rigorous approach
                – Bill Dubuque
                10 hours ago








              • 1




                @BillDubuque, it's not difficult to make it rigorous, to prove that the quotient $q$ can't be bigger than $3$. $(2q-7)B^2+(9q-9)B+(7q-2)$ is strictly positive if $qge 4$ and $B>0$. Hence if $qge 4$, no positive $B$ solves the equation. Similarly, if $q=1$, then the only positive solution is $B=1$, which is not possible.
                – vadim123
                9 hours ago












              • @vadim You should add the details of a rigorous proof to the answer (I've lost count of the number of times "[this case] doesn't make sense" turned out to be incorrect), so we should not encourage students to handwave like that.
                – Bill Dubuque
                9 hours ago
















              10












              10








              10






              The long division is the source of the error; you can't have $7/2$ as the quotient. The quotient needs to be an integer, that's what "factor" means.



              If the quotient is $2$, then the base is $4$. This is found by solving $7B^2+9B+2=color{red}{ 2}(2B^2+9B+7)$, and discarding the negative root.



              If the quotient is $3$, then the base is $19$. This is found by solving $7B^2+9B+2=color{red}{ 3}(2B^2+9B+7)$, and discarding the negative root.



              No other quotients make any sense. However, if the base is $4$, then you don't get digits $7$ and $9$. Hence the answer must be $B=19$.






              share|cite|improve this answer












              The long division is the source of the error; you can't have $7/2$ as the quotient. The quotient needs to be an integer, that's what "factor" means.



              If the quotient is $2$, then the base is $4$. This is found by solving $7B^2+9B+2=color{red}{ 2}(2B^2+9B+7)$, and discarding the negative root.



              If the quotient is $3$, then the base is $19$. This is found by solving $7B^2+9B+2=color{red}{ 3}(2B^2+9B+7)$, and discarding the negative root.



              No other quotients make any sense. However, if the base is $4$, then you don't get digits $7$ and $9$. Hence the answer must be $B=19$.







              share|cite|improve this answer












              share|cite|improve this answer



              share|cite|improve this answer










              answered 10 hours ago









              vadim123

              75.5k897189




              75.5k897189












              • Thank you so much. This was really very helpful. :)
                – Aamir Khan
                10 hours ago










              • My pleasure, glad to help.
                – vadim123
                10 hours ago










              • @AamirKhan Beware that the above is not a rigorous solution without proof that those are the only possible quotients. See my answer for one rigorous approach
                – Bill Dubuque
                10 hours ago








              • 1




                @BillDubuque, it's not difficult to make it rigorous, to prove that the quotient $q$ can't be bigger than $3$. $(2q-7)B^2+(9q-9)B+(7q-2)$ is strictly positive if $qge 4$ and $B>0$. Hence if $qge 4$, no positive $B$ solves the equation. Similarly, if $q=1$, then the only positive solution is $B=1$, which is not possible.
                – vadim123
                9 hours ago












              • @vadim You should add the details of a rigorous proof to the answer (I've lost count of the number of times "[this case] doesn't make sense" turned out to be incorrect), so we should not encourage students to handwave like that.
                – Bill Dubuque
                9 hours ago




















              • Thank you so much. This was really very helpful. :)
                – Aamir Khan
                10 hours ago










              • My pleasure, glad to help.
                – vadim123
                10 hours ago










              • @AamirKhan Beware that the above is not a rigorous solution without proof that those are the only possible quotients. See my answer for one rigorous approach
                – Bill Dubuque
                10 hours ago








              • 1




                @BillDubuque, it's not difficult to make it rigorous, to prove that the quotient $q$ can't be bigger than $3$. $(2q-7)B^2+(9q-9)B+(7q-2)$ is strictly positive if $qge 4$ and $B>0$. Hence if $qge 4$, no positive $B$ solves the equation. Similarly, if $q=1$, then the only positive solution is $B=1$, which is not possible.
                – vadim123
                9 hours ago












              • @vadim You should add the details of a rigorous proof to the answer (I've lost count of the number of times "[this case] doesn't make sense" turned out to be incorrect), so we should not encourage students to handwave like that.
                – Bill Dubuque
                9 hours ago


















              Thank you so much. This was really very helpful. :)
              – Aamir Khan
              10 hours ago




              Thank you so much. This was really very helpful. :)
              – Aamir Khan
              10 hours ago












              My pleasure, glad to help.
              – vadim123
              10 hours ago




              My pleasure, glad to help.
              – vadim123
              10 hours ago












              @AamirKhan Beware that the above is not a rigorous solution without proof that those are the only possible quotients. See my answer for one rigorous approach
              – Bill Dubuque
              10 hours ago






              @AamirKhan Beware that the above is not a rigorous solution without proof that those are the only possible quotients. See my answer for one rigorous approach
              – Bill Dubuque
              10 hours ago






              1




              1




              @BillDubuque, it's not difficult to make it rigorous, to prove that the quotient $q$ can't be bigger than $3$. $(2q-7)B^2+(9q-9)B+(7q-2)$ is strictly positive if $qge 4$ and $B>0$. Hence if $qge 4$, no positive $B$ solves the equation. Similarly, if $q=1$, then the only positive solution is $B=1$, which is not possible.
              – vadim123
              9 hours ago






              @BillDubuque, it's not difficult to make it rigorous, to prove that the quotient $q$ can't be bigger than $3$. $(2q-7)B^2+(9q-9)B+(7q-2)$ is strictly positive if $qge 4$ and $B>0$. Hence if $qge 4$, no positive $B$ solves the equation. Similarly, if $q=1$, then the only positive solution is $B=1$, which is not possible.
              – vadim123
              9 hours ago














              @vadim You should add the details of a rigorous proof to the answer (I've lost count of the number of times "[this case] doesn't make sense" turned out to be incorrect), so we should not encourage students to handwave like that.
              – Bill Dubuque
              9 hours ago






              @vadim You should add the details of a rigorous proof to the answer (I've lost count of the number of times "[this case] doesn't make sense" turned out to be incorrect), so we should not encourage students to handwave like that.
              – Bill Dubuque
              9 hours ago













              6














              Going $1$ step more with Euclid's algorithm reveals a common factor $,b!+!1.,$ Cancelling it



              $$dfrac{7b^2!+!9b!+!2}{2b^2!+!9b!+!7} = color{#c00}{dfrac{7b!+!2}{2b!+!7}}inBbb Z , Rightarrow, 7-2 dfrac{color{#c00}{7b!+!2}}{ color{#c00}{2b!+!7}}, =, dfrac{45}{2b!+!7}inBbb Zqquad$$



              Therefore $,2b!+!7mid 45 $ so $,b> 9,$(= digit) $,Rightarrow,2b!+!7 = 45,$ $Rightarrow,b=19.$






              share|cite|improve this answer




























                6














                Going $1$ step more with Euclid's algorithm reveals a common factor $,b!+!1.,$ Cancelling it



                $$dfrac{7b^2!+!9b!+!2}{2b^2!+!9b!+!7} = color{#c00}{dfrac{7b!+!2}{2b!+!7}}inBbb Z , Rightarrow, 7-2 dfrac{color{#c00}{7b!+!2}}{ color{#c00}{2b!+!7}}, =, dfrac{45}{2b!+!7}inBbb Zqquad$$



                Therefore $,2b!+!7mid 45 $ so $,b> 9,$(= digit) $,Rightarrow,2b!+!7 = 45,$ $Rightarrow,b=19.$






                share|cite|improve this answer


























                  6












                  6








                  6






                  Going $1$ step more with Euclid's algorithm reveals a common factor $,b!+!1.,$ Cancelling it



                  $$dfrac{7b^2!+!9b!+!2}{2b^2!+!9b!+!7} = color{#c00}{dfrac{7b!+!2}{2b!+!7}}inBbb Z , Rightarrow, 7-2 dfrac{color{#c00}{7b!+!2}}{ color{#c00}{2b!+!7}}, =, dfrac{45}{2b!+!7}inBbb Zqquad$$



                  Therefore $,2b!+!7mid 45 $ so $,b> 9,$(= digit) $,Rightarrow,2b!+!7 = 45,$ $Rightarrow,b=19.$






                  share|cite|improve this answer














                  Going $1$ step more with Euclid's algorithm reveals a common factor $,b!+!1.,$ Cancelling it



                  $$dfrac{7b^2!+!9b!+!2}{2b^2!+!9b!+!7} = color{#c00}{dfrac{7b!+!2}{2b!+!7}}inBbb Z , Rightarrow, 7-2 dfrac{color{#c00}{7b!+!2}}{ color{#c00}{2b!+!7}}, =, dfrac{45}{2b!+!7}inBbb Zqquad$$



                  Therefore $,2b!+!7mid 45 $ so $,b> 9,$(= digit) $,Rightarrow,2b!+!7 = 45,$ $Rightarrow,b=19.$







                  share|cite|improve this answer














                  share|cite|improve this answer



                  share|cite|improve this answer








                  edited 3 hours ago

























                  answered 10 hours ago









                  Bill Dubuque

                  208k29190628




                  208k29190628























                      1














                      Since $b+1>0$ and $$(b+1)(2b+7)mid (7b+2)(b+1)implies 2b+7mid 7b+2$$



                      we have $$2b+7mid (7b+2)-3(2b+7) = b-19$$



                      so if $b-19> 0$ we have $$2b+7mid b-19 implies 2b+7leq b-19 implies b+26leq 0$$



                      which is not true. So $bleq 19$. By trial and error we see that $b=4$ and $b=19$ works.






                      share|cite|improve this answer























                      • This answer is of no use?
                        – greedoid
                        7 hours ago
















                      1














                      Since $b+1>0$ and $$(b+1)(2b+7)mid (7b+2)(b+1)implies 2b+7mid 7b+2$$



                      we have $$2b+7mid (7b+2)-3(2b+7) = b-19$$



                      so if $b-19> 0$ we have $$2b+7mid b-19 implies 2b+7leq b-19 implies b+26leq 0$$



                      which is not true. So $bleq 19$. By trial and error we see that $b=4$ and $b=19$ works.






                      share|cite|improve this answer























                      • This answer is of no use?
                        – greedoid
                        7 hours ago














                      1












                      1








                      1






                      Since $b+1>0$ and $$(b+1)(2b+7)mid (7b+2)(b+1)implies 2b+7mid 7b+2$$



                      we have $$2b+7mid (7b+2)-3(2b+7) = b-19$$



                      so if $b-19> 0$ we have $$2b+7mid b-19 implies 2b+7leq b-19 implies b+26leq 0$$



                      which is not true. So $bleq 19$. By trial and error we see that $b=4$ and $b=19$ works.






                      share|cite|improve this answer














                      Since $b+1>0$ and $$(b+1)(2b+7)mid (7b+2)(b+1)implies 2b+7mid 7b+2$$



                      we have $$2b+7mid (7b+2)-3(2b+7) = b-19$$



                      so if $b-19> 0$ we have $$2b+7mid b-19 implies 2b+7leq b-19 implies b+26leq 0$$



                      which is not true. So $bleq 19$. By trial and error we see that $b=4$ and $b=19$ works.







                      share|cite|improve this answer














                      share|cite|improve this answer



                      share|cite|improve this answer








                      edited 9 hours ago

























                      answered 9 hours ago









                      greedoid

                      37.9k114794




                      37.9k114794












                      • This answer is of no use?
                        – greedoid
                        7 hours ago


















                      • This answer is of no use?
                        – greedoid
                        7 hours ago
















                      This answer is of no use?
                      – greedoid
                      7 hours ago




                      This answer is of no use?
                      – greedoid
                      7 hours ago











                      0














                      $$2B^2+9B+7mid 7B^2+9B+2$$



                      Let's write $aB^2+bB + c$ as $[a,b,c]_B$ to emphasis that $a,b,c$ are digits base $B$.



                      Then $[2,9,7]_B mid [7,9,2]_B-[2,9,7]_B$ and we are assuming that $2,9,7 < B$



                      Writing this out "subtraction-style", we get



                      $left.begin{array}{c}
                      7 & 9 & 2 \
                      2 & 9 & 7 \
                      hline
                      phantom{4}
                      end{array}
                      right.
                      implies
                      left.begin{array}{c}
                      6 & (B+8) & (B+2) \
                      2 & 9 & 7 \
                      hline
                      4 & (B-1) & (B-5)
                      end{array}
                      right.
                      $



                      So $[4,B-1,B-5]_B$ is a multiple of $[2,9,7]_B$.



                      We must therefore have $[4,B-1,B-5]_B = 2[2,9,7]_B = [4,18,14]_B$ which implies $B-1=18$ and $B-5=14$. Hence $B=19$.






                      share|cite|improve this answer




























                        0














                        $$2B^2+9B+7mid 7B^2+9B+2$$



                        Let's write $aB^2+bB + c$ as $[a,b,c]_B$ to emphasis that $a,b,c$ are digits base $B$.



                        Then $[2,9,7]_B mid [7,9,2]_B-[2,9,7]_B$ and we are assuming that $2,9,7 < B$



                        Writing this out "subtraction-style", we get



                        $left.begin{array}{c}
                        7 & 9 & 2 \
                        2 & 9 & 7 \
                        hline
                        phantom{4}
                        end{array}
                        right.
                        implies
                        left.begin{array}{c}
                        6 & (B+8) & (B+2) \
                        2 & 9 & 7 \
                        hline
                        4 & (B-1) & (B-5)
                        end{array}
                        right.
                        $



                        So $[4,B-1,B-5]_B$ is a multiple of $[2,9,7]_B$.



                        We must therefore have $[4,B-1,B-5]_B = 2[2,9,7]_B = [4,18,14]_B$ which implies $B-1=18$ and $B-5=14$. Hence $B=19$.






                        share|cite|improve this answer


























                          0












                          0








                          0






                          $$2B^2+9B+7mid 7B^2+9B+2$$



                          Let's write $aB^2+bB + c$ as $[a,b,c]_B$ to emphasis that $a,b,c$ are digits base $B$.



                          Then $[2,9,7]_B mid [7,9,2]_B-[2,9,7]_B$ and we are assuming that $2,9,7 < B$



                          Writing this out "subtraction-style", we get



                          $left.begin{array}{c}
                          7 & 9 & 2 \
                          2 & 9 & 7 \
                          hline
                          phantom{4}
                          end{array}
                          right.
                          implies
                          left.begin{array}{c}
                          6 & (B+8) & (B+2) \
                          2 & 9 & 7 \
                          hline
                          4 & (B-1) & (B-5)
                          end{array}
                          right.
                          $



                          So $[4,B-1,B-5]_B$ is a multiple of $[2,9,7]_B$.



                          We must therefore have $[4,B-1,B-5]_B = 2[2,9,7]_B = [4,18,14]_B$ which implies $B-1=18$ and $B-5=14$. Hence $B=19$.






                          share|cite|improve this answer














                          $$2B^2+9B+7mid 7B^2+9B+2$$



                          Let's write $aB^2+bB + c$ as $[a,b,c]_B$ to emphasis that $a,b,c$ are digits base $B$.



                          Then $[2,9,7]_B mid [7,9,2]_B-[2,9,7]_B$ and we are assuming that $2,9,7 < B$



                          Writing this out "subtraction-style", we get



                          $left.begin{array}{c}
                          7 & 9 & 2 \
                          2 & 9 & 7 \
                          hline
                          phantom{4}
                          end{array}
                          right.
                          implies
                          left.begin{array}{c}
                          6 & (B+8) & (B+2) \
                          2 & 9 & 7 \
                          hline
                          4 & (B-1) & (B-5)
                          end{array}
                          right.
                          $



                          So $[4,B-1,B-5]_B$ is a multiple of $[2,9,7]_B$.



                          We must therefore have $[4,B-1,B-5]_B = 2[2,9,7]_B = [4,18,14]_B$ which implies $B-1=18$ and $B-5=14$. Hence $B=19$.







                          share|cite|improve this answer














                          share|cite|improve this answer



                          share|cite|improve this answer








                          edited 3 hours ago

























                          answered 5 hours ago









                          steven gregory

                          17.7k32257




                          17.7k32257






























                              draft saved

                              draft discarded




















































                              Thanks for contributing an answer to Mathematics Stack Exchange!


                              • Please be sure to answer the question. Provide details and share your research!

                              But avoid



                              • Asking for help, clarification, or responding to other answers.

                              • Making statements based on opinion; back them up with references or personal experience.


                              Use MathJax to format equations. MathJax reference.


                              To learn more, see our tips on writing great answers.





                              Some of your past answers have not been well-received, and you're in danger of being blocked from answering.


                              Please pay close attention to the following guidance:


                              • Please be sure to answer the question. Provide details and share your research!

                              But avoid



                              • Asking for help, clarification, or responding to other answers.

                              • Making statements based on opinion; back them up with references or personal experience.


                              To learn more, see our tips on writing great answers.




                              draft saved


                              draft discarded














                              StackExchange.ready(
                              function () {
                              StackExchange.openid.initPostLogin('.new-post-login', 'https%3a%2f%2fmath.stackexchange.com%2fquestions%2f3058019%2ffactors-in-a-different-base-2b2-9b-7-mid-7b2-9b-2%23new-answer', 'question_page');
                              }
                              );

                              Post as a guest















                              Required, but never shown





















































                              Required, but never shown














                              Required, but never shown












                              Required, but never shown







                              Required, but never shown

































                              Required, but never shown














                              Required, but never shown












                              Required, but never shown







                              Required, but never shown







                              Popular posts from this blog

                              Quarter-circle Tiles

                              build a pushdown automaton that recognizes the reverse language of a given pushdown automaton?

                              Mont Emei